Diễn Đàn MathScope

Diễn Đàn MathScope (http://forum.mathscope.org/index.php)
-   Giải Tích/Analysis (http://forum.mathscope.org/forumdisplay.php?f=13)
-   -   Qua giới hạn dưới dấu tích phân (http://forum.mathscope.org/showthread.php?t=51989)

portgas_d_ace 13-02-2019 02:11 PM

Qua giới hạn dưới dấu tích phân
 
Mình gặp một vấn đề khi đọc quyển Một số pp giải bài toán biên phi tuyến của cụ J. L. Lions.
Cụ thể như sau: cho $h \in {L^1}\left( {0,T} \right)$, chứng minh rằng với hầu hết $t \in \left( {0,T} \right)$ ta đều có
\[\mathop {\lim }\limits_{n \to + \infty } n\int\limits_t^{t + \frac{1}{n}} {\left[ {1 - n\left( {s - t} \right)} \right]h\left( s \right)ds} = \frac{1}{2}h\left( t \right).\]
Cụ Lions bảo dùng định lý Lebesgue nhưng rõ là hội tụ bị chặn của Lebesgue không dùng được. Mình nghĩ là dùng định lý về điểm Lebesgue nhưng vẫn chưa ra. =P~

123456 19-02-2019 11:29 PM

Trích:

Nguyên văn bởi portgas_d_ace (Post 214038)
Mình gặp một vấn đề khi đọc quyển Một số pp giải bài toán biên phi tuyến của cụ J. L. Lions.
Cụ thể như sau: cho $h \in {L^1}\left( {0,T} \right)$, chứng minh rằng với hầu hết $t \in \left( {0,T} \right)$ ta đều có
\[\mathop {\lim }\limits_{n \to + \infty } n\int\limits_t^{t + \frac{1}{n}} {\left[ {1 - n\left( {s - t} \right)} \right]h\left( s \right)ds} = \frac{1}{2}h\left( t \right).\]
Cụ Lions bảo dùng định lý Lebesgue nhưng rõ là hội tụ bị chặn của Lebesgue không dùng được. Mình nghĩ là dùng định lý về điểm Lebesgue nhưng vẫn chưa ra. =P~

Mở rộng hàm $h$ cho nó bằng $0$ bên ngoài đoạn $(0,T)$, khi đó $h \in L^1(\mathbb R)$. Ta chứng minh rẳng với hầu hết $t\in \mathbb R$, ta có
\[ \lim_{n \to \infty} n \int_t^{t + \frac 1n} (1 -n(s-t)) h(s) ds = \frac12 h(t).\]
Thật vậy, xét hàm
\[ \varphi_n(t) = n \int_t^{t + \frac 1n} (1 -n(s-t)) h(s) ds = \int_0^1 (1-u) h\left(t+ \frac u n\right) du.\]
Ta có
\[ \varphi_n(t) - \frac 12 h(t) = \int_0^1 (1-u)\left[ h\left(t+ \frac u n\right) - h(t)\right] du,
\]
do đó
\[ \left| \varphi_n(t) - \frac 12 h(t)\right| \leq \int_0^1 (1-u)\left|h\left(t+ \frac u n\right) - h(t)\right| du.
\]
Tích phân 2 vế trên $\mathbb R$ và dùng định lý Fubini, ta được
\[
\int_{\mathbb R} \left| \varphi_n(t) - \frac 12 h(t)\right| dt \leq \int_0^1 (1-u) \int_{\mathbb R} \left|h\left(t+ \frac u n\right) - h(t)\right| dt du.
\]
Do chuẩn $L^1$ liên tục với phép tịnh tiến nên
\[
\lim_{n\to \infty } \int_{\mathbb R} \left|h\left(t+ \frac u n\right) - h(t)\right| dt = 0,
\]
với mọi $u \in [0,1]$. Mặt khác ta có
\[
\int_{\mathbb R} \left|h\left(t+ \frac u n\right) - h(t)\right| dt \leq 2 \int_{\mathbb R} |h(t)| dt.
\]
Sử dụng định lý hội tụ bị chặn của Lebesgue, ta được
\[
\lim_{n\to \infty} \int_{\mathbb R} \left| \varphi_n(t) - \frac 12 h(t)\right| dt =0.
\]
Do đó, với hầu hết $t \in \mathbb R$ ta có $\varphi_n(t) \to \frac 12 h(t)$. Đặt $A$ là tập các $t$ như vậy. Khi đó $B = \mathbb R \setminus A$ có độ đo $0$. Do đó $B \cap (0,T)$ có độ đo $0$. Hiển nhiên với mọi $t \in (0,T) \setminus B$ thì $t \in A$ do đó $\varphi_n(t) \to \frac 12 h(t)$.

portgas_d_ace 20-02-2019 11:26 AM

Em trình bày một cách khác như sau
\[n\int\limits_t^{t + \frac{1}{n}} {\left[ {1 - n\left( {s - t} \right)} \right]h\left( s \right)ds} = n\int\limits_t^{t + \frac{1}{n}} {h\left( s \right)ds} - {n^2}\int\limits_t^{t + \frac{1}{n}} {\left( {s - t} \right)h\left( s \right)ds} .\]
Do định lý về điểm Lebesgue, ta có
\[\mathop {\lim }\limits_{n \to + \infty } n\int\limits_t^{t + \frac{1}{n}} {h\left( s \right)ds} = h\left( t \right).\]
Nên ta chỉ cần chứng minh
\[\mathop {\lim }\limits_{n \to + \infty } {n^2}\int\limits_t^{t + \frac{1}{n}} {\left( {s - t} \right)h\left( s \right)ds} = \frac{1}{2}h\left( t \right).\]
Ta có
\[\left| {{n^2}\int\limits_t^{t + \frac{1}{n}} {\left( {s - t} \right)h\left( s \right)ds} - \frac{1}{2}h\left( t \right)} \right| = \left| {{n^2}\int\limits_t^{t + \frac{1}{n}} {\left( {s - t} \right)\left( {h\left( s \right) - h\left( t \right)} \right)ds} } \right| \leqslant n\int\limits_t^{t + \frac{1}{n}} {\left| {h\left( s \right) - h\left( t \right)} \right|ds} .\]
Lại do định lý về điểm Lebesgue, ta có
\[\mathop {\lim }\limits_{n \to + \infty } n\int\limits_t^{t + \frac{1}{n}} {\left| {h\left( s \right) - h\left( t \right)} \right|ds} = 0.\]
Vậy
\[\mathop {\lim }\limits_{n \to + \infty } {n^2}\int\limits_t^{t + \frac{1}{n}} {\left( {s - t} \right)h\left( s \right)ds} = \frac{1}{2}h\left( t \right).\]
Chứng minh hoàn tất.:sexygirl:


Múi giờ GMT. Hiện tại là 09:26 PM.

Powered by: vBulletin Copyright ©2000-2024, Jelsoft Enterprises Ltd.

[page compression: 8.03 k/8.40 k (4.38%)]